You are on page 1of 40

2016_ADVANCE_PAPER-2

PHYSICS
1. The electrostatic energy of Z protons uniformly distributed
throughout a spherical nucleus of radius R is given by:
3 Z ( Z − 1) e
2

E=
5 4πε 0 R

7 N and 8 O are
The measured masses of the neutron, 11 H, 15 15

1.008665 u, 1.007825 u, 15.000109 u and 15.003065u,


respectively. Given that the radii of both the 15
7 N and 8 O nuclei
15

are same, 1 u = 931.5 MeV/c2 (c is the speed of light) and


e2/(4επ0) = 1.44 MeV fm. Assuming that the difference

7 N and 8 O is purely due to


between the binding energies of 15 15

the electrostatic energy, the radius of either of the nuclei is

(1 fm = 10 −15
m)

(A) 2.85 fm
(B) 3.03 fm
(C) 3.42 fm
(D) 3.80 fm

Advanced Paper-2
2. An accident in a nuclear laboratory resulted in deposition of a
certain amount of radioactive material of half-life 18 days
inside the laboratory. Tests revealed that the radiation was 64
times more than the permissible level required for safe
operation of the laboratory. What is the minimum number of
days after which the laboratory can be considered safe for use?:
(A) 64
(B) 90
(C) 108
(D) 120

3. A gas is enclosed in a cylinder with a movable frictionless


piston. Its initial thermodynamic state at pressure Pi = 105 Pa
and volume Vi = 10-3 m3 changes to a final state at Pf = (1/32) ×
105 Pa and Vf = 8 × 10-3 m3 in an adiabatic quasi-static process,
such that P3V5 = constant. Consider another thermodynamic
process that brings the system from the same initial state to the
same final state in two steps: an isobaric expansion at Pi
followed by an isochoric (isovolumetric) process at volume Vf.
The amount of heat supplied to the system in the two-step
process is approximately:

Advanced Paper-2
(A) 112 J
(B) 294 J
(C) 588 J
(D) 813 J

4. The ends Q and R of two thin wires, PQ and RS, are soldered
(joined) together. Initially each of the wires has a length of 1 m
at 10 0C. Now the end P is maintained at 10 0C, while the end S
is heated and maintained at 400 0C. The system is thermally
insulated from its surroundings. If the thermal conductivity of
wire PQ is twice that of the wire RS and the coefficient of
linear thermal expansion of PQ is 1.2 × 10-5 K-1, the change in
length of the wire PQ is:
(A) 0.78 mm
(B) 0.90 mm
(C) 1.56 mm
(D) 2.34 mm

5. A small object is placed 50 cm to the left of a thin convex lens


of focal length 30 cm. A convex spherical mirror of radius of
curvature 100 cm is placed to the right of the lens at a distance

Advanced Paper-2
of 50 cm. The mirror is tilted such that the axis of the mirror is
at an angle θ = 300 to the axis of the lens, as shown in the
figure.

If the origin of the coordinate system is taken to be at the


centre of the lens, the coordinates (in cm) of the point (x, y) at
which the image is formed are
(A) (0 , 0)
(B) (50-25 3 , 25)
(C) (25, 25 3 )
(D) (125/3, 25/ 3 )

6. There are two Vernier calipers both of which have 1 cm


divided into 10 equal divisions on the main scale. The Vernier
scale of one of the calipers (C1) has 10 equal divisions that
correspond to 9 main scale divisions. The Vernier scale of the
other caliper (C2) has 10 equal divisions that correspond to 11
Advanced Paper-2
main scale divisions. The readings of the two calipers are
shown in the figure. The measured values (in cm) by calipers
C1 and C2 respectively, are

(A) 2.85 and 2.82


(B) 2.87 and 2.83
(C) 2.87 and 2.86
(D) 2.87 and 2.87

7. Two thin circular discs of mass m and 4m, having radii of a


and 2a, respectively, are rigidly fixed by a massless, rigid rod
of length l = 24 a through their centers. This assembly is laid
on a firm and flat surface, and set rolling without slipping on
the surface so that the angular speed about the axis of the rod is
ω. The angular momentum of the entire assembly about the

Advanced Paper-2

point ‘O’ is L (see the figure). Which of the following
statement(s) is (are) true?

(A) The center of mass of the assembly rotates about the z-


axis with an angular speed of ω/5
(B) The magnitude of angular momentum of the assembly
about its center of mass is 17 ma2 ω/2
(C) The magnitude of angular momentum of center of mass
of the assembly about the point O is 81 ma2ω

(D) The magnitude of the z-component of L is 55 ma2ω

8. Light of wavelength λph falls on a cathode plate inside a


vacuum tube as shown in the figure. The work function of the
cathode surface is φ and the anode is a wire mesh of
conducting material kept at a distance d from the cathode. A
potential difference V is maintained between the electrodes. If
the minimum de Broglie wavelength of the electrons passing
Advanced Paper-2
through the anode is λe, which of the following statement(s) is
(are) true?:

(A) λe decreases with increase in φ and λph


(B) λe is approximately halved, if d is doubled
(C) For large potential difference (V >> φ/e), λe is
approximately halved if V is made four times
(D) λe increases at the same rate as λph for λph < hc/φ

9. In an experiment to determine the acceleration due to gravity g,


the formula used for the time period of a periodic motion is

7(R − r)
T = 2π . The values of R and r are measured to be (60
5g

± 1) mm and (10 ± 1) mm, respectively. In five successive


measurements, the time period is found to be 0.52 s, 0.56 s,
0.57 s, 0.54 s and 0.59 s. The least count of the watch used for

Advanced Paper-2
the measurement of time period is 0.01 s. Which of the
following statement(s) is (are) true?:
(A) The error in the measurement of r is 10%
(B) The error in the measurement of T is 3.57%
(C) The error in the measurement of T is 2%
(D) The error in the determined value of g is 11%

10. Consider two identical galvanometers and two identical


resistors with resistance R. If the internal resistance of the
galvanometers RC < R/2, which of the following statement(s)
about any one of the galvanometers is (are) true?
(A) The maximum voltage range is obtained when all the
components are connected in series
(B) The maximum voltage range is obtained when the two
resistors and one galvanometer are connected in
series, and the second galvanometer is connected in
parallel to the first galvanometer
(C) The maximum current range is obtained when all the
components are connected in parallel
(D) The maximum current range is obtained when the two
galvanometers are connected in series and the

Advanced Paper-2
combination is connected in parallel with both the
resistors

11. In the circuit shown below, the key is pressed at time t = 0.


Which of the following statement(s) is (are)
true?

(A) The voltmeter displays – 5 V as soon as the key is


pressed, and displays + 5 V after a long time
(B) The voltmeter will display 0 V at time t = ln 2 seconds
(C) The current in the ammeter becomes 1/e of the initial
value after 1 second
(D) The current in the ammeter becomes zero after a long
time

Advanced Paper-2
12. A block with mass M is connected by a massless spring with
stiffness constant k to a rigid wall and moves without friction
on a horizontal surface. The block oscillates with small
amplitude A about an equilibrium position x0. Consider two
cases: (i) when the block is at x0; and (ii) when the block is at x
= x0 + A. In both the cases, a particle with mass m (< M) is
softly placed on the block after which they stick to each other.
Which of the following statement(s) is (are) true about the
motion after the mass m is placed on the mass M ?
(A) The amplitude of oscillation in the first case changes by a
M
factor of , whereas in the second case it remains
m+M
unchanged
(B) The final time period of oscillation in both the cases is
same
(C) The total energy decreases in both the cases
(D) The instantaneous speed at x0 of the combined masses
decreases in both the cases

13. While conducting the Young’s double slit experiment, a


student replaced the two slits with a large opaque plate in the x-

Advanced Paper-2
y plane containing two small holes that act as two coherent
point sources (S1, S2) emitting light of wavelength 600 nm. The
student mistakenly placed the screen parallel to the x-z plane
(for z > 0) at a distance D = 3 m from the mid-point of S1S2, as
shown schematically in the figure. The distance between the
sources d = 0.6003 mm. The origin O is at the intersection of
the screen and the line joining S1S2. Which of the following is
(are) true of the intensity pattern on the screen?

(A) Straight bright and dark bands parallel to the x-axis


(B) The region very close to the point O will be dark
(C) Hyperbolic bright and dark bands with foci symmetrically
placed about O in the x-direction
(D) Semi circular bright and dark bands centred at point O

14. A rigid wire loop of square shape having side of length L and
resistance R is moving along the x-axis with a constant velocity
v0 in the plane of the paper. At t = 0, the right edge of the loop
Advanced Paper-2
enters a region of length 3L where there is a uniform magnetic
field B0 into the plane of the paper, as shown in the figure. For
sufficiently large v0, the loop eventually crosses the region. Let
x be the location of the right edge of the loop. Let v(x), I(x) and
F(x) represent the velocity of the loop, current in the loop, and
force on the loop, respectively, as a function of x. Counter-
clockwise current is taken as positive:

Which of the following schematic plot(s) is(are) correct?


(Ignore gravity)

(A)

Advanced Paper-2
(B)

(C)

(D)

PARAGRAPH 1
A frame of reference that is accelerated with respect to an
inertial frame of reference is called a non-inertial frame of
reference. A coordinate system fixed on a circular disc rotating
about a fixed axis with a constant angular velocity ω is an
example of a non-inertial frame of reference. The relationship

between the force Frot experienced by a particle of mass m

Advanced Paper-2

moving on the rotating disc and the force Fin experienced by
the particle in an inertial frame of reference is
      
Frot ( ) (
= Fin + 2m ν rot × ω + m ω× r × ω, )

Where ν rot is the velocity of the particle in the rotating frame

of reference and r is the position vector of the particle with
respect to the centre of the disc.
Now consider a smooth slot along a diameter of a disc of radius
R rotating counter-clockwise with a constant angular speed ω
about its vertical axis through its center. We assign a
coordinate system with the origin at the center of the disc, the
x-axis along the slot, the y-axis perpendicular to the slot and the
 
( )
z-axis along the rotation axis ω = ωk . A small block of mass

m is gently placed in the slot at r = ( R 2 ) ˆi at t = 0 and is

constrained to move only along the slot.

Advanced Paper-2
15. The distance r of the block at time t is
R ωt
(A)
4
( e + e −ωt )

R
(B) cos ωt
2
R 2 ωt −2 ωt
(C)
4
(e + e )
R
(D) cos 2ωt
2

16. The net reaction of the disc on the block is:


1
(A) mω2 R ( e 2 ωt − e −2 ωt ) ˆj + mgkˆ
2
1
(B) mω2 R ( eωt − e −ωt ) ˆj + mgkˆ
2
(C) − mω2 R cos ωtjˆ − mgkˆ

(D) mω2 R sin ωtjˆ − mgkˆ

PARAGRAPH 2
Consider an evacuated cylindrical chamber of height h having
rigid conducting plates at the ends and an insulating curved
surface as shown in the figure. A number of spherical balls

Advanced Paper-2
made of a light weight and soft material and coated with a
conducting material are placed on the bottom plate. The balls
have a radius r << h. Now a high voltage source (HV) is
connected across the conducting plates such that the bottom
plate is at +V0 and the top plate at –V0. Due to their conducting
surface, the balls will get charged, will become equipotential
with the plate and are repelled by it. The balls will eventually
collide with the top plate, where the coefficient of restitution
can be taken to be zero due to the soft nature of the material of
the balls. The electric field in the chamber can be considered to
be that of a parallel plate capacitor. Assume that there are no
collisions between the balls and the interaction between them is
negligible. (Ignore gravity)

17. Which one of the following statements is correct?


(A) The balls will stick to the top plate and remain there

Advanced Paper-2
(B) The balls will bounce back to the bottom plate carrying
the same charge they went up with
(C) The balls will bounce back to the bottom plate carrying
the opposite charge they went up with
(D) The balls will execute simple harmonic motion between
the two plates

18. The average current in the steady state registered by the


ammeter in the circuit will be
(A) Zero
(B) proportional to the potential V0
(C) proportional to V01 2
(D) proportional to V02

Advanced Paper-2
2016_Advance_Paper-2
Answer Key

1 * 7 A OR A, C 13 B, D
2 C 8 C 14 A, B
3 C 9 A, B, D 15 A
4 A 10 A, C 16 B
5 C 11 A, B, C, D 17 C
6 B 12 A, B, D 18 D
19 D 28 A,B and C
20 D 29 B and D
21 A 30 B and C
22 A 31 C and D
23 A 32 B,C and D
24 A 33 B
25 A and C 34 C
26 A and B 35 A
27 B,C and 36 B
D
37. B 38. B 39. C 40. A
41. C 42. C 43. B, C 44. A, B
45. A, D 46. B, C 47. A, C, D 48. A, C, D
Advanced Paper-2
49. B, C, D 50. B, C 51. B 52. C
53. A 54. C

Advanced Paper-2
PHYSICS SOLUTION

1. Sol:
The formula for energy is,
3 Z ( Z − 1) e
2

E=
5 4πε 0 R

So, binding energy of oxygen is,


3 ( 8 )( 8 − 1) e
2

EO =
5 4πε 0 R
3 ( 8 )( 7 )
= (1.44 )
5 R
Similarly, for binding energy of nitrogen is,
3 ( 7 )( 7 − 1) e
2

EN =
5 4πε 0 R
3 ( 7 )( 6 )
= (1.44 )
5 R
The difference of electrostatic energies,
3 ( 2 )( 6 )
EO − EN = (1.44 )
5 R …… (1)
10.368
= MeV ⋅ fm
R
The binding energy of O atoms is,
BO = 8mp + 7 mn − AO

The binding energy of N atoms is,


BN = 7 mp + 8mn − AN

The difference of binding energies,


BO − BN = mp − mn − ( AO − AN )
= 1.007825 − 1.008665 − (15.003065 − 15.000109 )
= 0.003796 × 931.5
= 3.535974 MeV
Thus, from equation (1),
10.368MeV fm
R=
3.535974 MeV
= 2.932 fm
Advanced Paper-2
2. Sol:
The activity of the particle in radioactivity is,
Initial Activity
Required Activity =
64
= ( Initial Activity ) 2−6

The exponent of 2 is six, so the required time is six times the


half life.
Hence, the required time is,
Time required = 6T0.5
= 6 ×18 days
=108 days

3. Sol:
5
γ=
Since, 3 hence, the gas is monoatomic.
The work done is given by,
W= Pi ∆V
= 105 ( 8 ×10−3 − 10−3 )
= 700 J

The internal energy is,

Advanced Paper-2
∆U = nCv ∆T
3
= [ PV i i]
f f − PV
2
3 1 
=  ×105 × 8 ×10−3 − 105 ×10−3 
2  32 
900
= − J
8

From the First law of thermodynamics,


H= W + ∆U
900
= 700 −
8
5600 − 900
=
8
4700
=
8

Simplify the above equation.


H = 588 J

4. Sol:
The required diagram is shown below,

RPQ 1
=
RRS 2
400 − T
T − 10 =
2
3T
= 210
2
T = 140 C

As a function of x ,
Advanced Paper-2
T ( x=
) 10 + 130 x
∆T ( x =
) T ( x ) − 10= 130 x

Extension in small element of length dx ,

dl = α∆T ( x ) dx
= 130α xdx

Net extension,
1
∆l =130α ∫ xdx
0

130
= ×1.2 ×10−5 ×1
2
= 0.78 mm

5. Sol:
The required diagram is shown below,

The first image from the lens will be formed at 75 cm to the right
of the lens.
Take the mirror to be straight, then the image after reflection
forms at 50 cm to the left of the mirror.
Rotate the mirror by 30 , the final image is,

Advanced Paper-2
x 50 − 50 cos 60
=
= 50 − 25
= 25 cm

And,
y = 50sin 60
= 25 3 cm

6. Sol:
In first scale;
Least count of main scale is 1 mm .
10 VSD = 9 MSD

So,
Least count of vernier scale is,
0.9 mm

The main scale reading is,


2.8 cm

The vernier scale reading is,


1
7× 0.07 cm
=
10

So reading is,
2.8 + 0.07 =
2.87 cm

In second scale, the vernier scale is abnormal,

Advanced Paper-2
Distance measured from main scale is equal to the distance
measured from vernier scale.
(1)(8)
28+ = ( 28+x ) + (1.1)( 7 )
8= x + 7.7
x = 0.3

So reading is,
2.8 + 0.03 =
2.83 mm

7. Sol:
Evaluate the angular momentum in the given figure.

By Pythagoras theorem, the horizontal distance is calculated as,


2
X=
1 a 2 + L2
= a 2 + 24a 2
= 25a 2
X 1 = 5a

Angular velocity of centre of mass with respect to point O is,



Ω=
l

Angular velocity of centre of mass with respect to z axis is,

Advanced Paper-2

Ω= cos θ
l
aω l
= ×
l X1

=
5a
ω
=
5

Angular momentum about its centre is,


L = Iω
 ma 2 4m ( 2a )2 
=  + ω
 2 2 
17 ma 2
= ω
2

8. Sol:
Energy equation is,
hc hc
=φ + + eV
λph λe
 −1   −1 
hc  2 d λph  = hc  2 d λe 
λ 
 ph   λe 
d λph d λe
= 2
λph
2
λe
d λe λe d λph
=
λe λph λph

λe
λph
The wavelength λ increases e times the increase in
wavelength λph
.
For V >> φ e

Advanced Paper-2
hc
= eV
λ
h2
= eV
2mλe 2
1
λe ∝
V

Hence, to obtain half the wavelength, the voltage has to be four


times.

9. Sol:
The measured value of r is,
r ± ∆=
r (10 ± 1) mm

The relative error is


∆r 1
×100 = ×100
r 10
= 10%

The table for the given value is,


S.No. T T-Tmean

1 0.52 0.04
2 0.56 0.00
3 0.57 0.01
4 0.54 0.02
5 0.59 0.03
The formula for mean time period is,

Advanced Paper-2
ΣT
Tmean =
N
2.78
=
5
= 0.56

Σ T − Tmean
%error= ×100
5Tmean
0.1
= ×100
5 × 0.56
= 3.57%

According to the question,


7(R − r)
T = 2π
5g

So,
T 2 ∝ g (R − r)
T2
g∝
(R − r)

Therefore,
 dg  dT d ( R − r )
=  2 +
 g  T (R − r)
3.57 1+1
= 2× +
100 60 − 10
7.14 2
= −
50 50
5.14
=
50

Simplify the above equation.


 dg 
  ≈ 11%
 g 

Advanced Paper-2
10. Sol:
For maximum allowable current through the galvanometers, if
all the elements are in series, the voltage will be
V1 = ig Req
V=
1 ig ( R + R + Rg + Rg )
= 2ig ( R + Rg )

If the galvanometers are in parallel and the resistors are in


series.
=V2 ig ( RC  RC + R + R )
R 
= ig  C + 2 R 
 2 
ig
= ( RC + 4 R )
2

Divide V by V 1 2

V1 2ig ( R + RC )
=
V2 ig
( RC + 4 R )
2
4 ( R + RC )
=
RC + 4 R
4 R + 4 RC
=
RC + 4 R
3RC + 4 R + RC
=
RC + 4 R

Further solve,

Advanced Paper-2
V1 3RC
= +1
V2 RC + 4 R
3RC
= +1
 R 
RC  1 + 4 
 RC 
3
= +1
R
1+ 4
RC

Thus,
V1 > V2

Hence, maximum voltage range is when all the elements are in


series.
When all the elements are in parallel, the current is maximum
because least resistance is obtained when all the elements are in
parallel when compared to any other possibilities.

11. Sol:
Consider the diagram.

Advanced Paper-2
At t =0 (as soon as the key is pressed), the voltage across the
capacitors is zero, as capacitors oppose instantaneous change in
voltage. The voltmeter will read −5 V .

At t= ∞ (after a long time), the capacitors are completely


charged. The capacitors provide zero opposition when
completely charged. So, the voltmeter reading will be 5 V .
Hence, option (A) is correct.
At transient state,
t
5 − 50×103 ×20×10−6
I1 = e
50
1
= e−t
10

The value of I2 is calculated as,


t
5 − 25×103 ×40×10−6
I2 = e
25
1
= e−t
10

So, net current,


I= I1 + I 2
e−t e−t
= +
10 5
= 0.3e − t

So, the current becomes 1 e times the initial current after1 s


.Hence, option (C) is correct.
Also,

Advanced Paper-2
At t → ∞ , I → 0. Hence, the current in the ammeter becomes zero
after a long time. Hence, option (D) is correct.
The reading of voltmeter at any instant is,
V = ∆V40 + ∆V20
0 = 5 (1 − e − t ) − 5e − t
0 = 1 − 2e − t
t = ln 2

Hence, option (B) is correct.

12. Sol:
Consider the diagram.

Amplitude in case 1,
MV
=1 ( M + m )V2
 M 
V2 =   V1
M +m
k  M  k
A2 =  A1
M +m M +m M
M
A2 = A1
m+M

Amplitude in case 2,
Advanced Paper-2
A2 = A1

Time period of oscillations remains same for both cases,


M +m
T = 2π
k

The total energy in case 1,


1
E2 = kA2 2
2
1  M  2
= k  A1
2 m+M 
 M 
=  E1
m+M 

The total energy in case 2,


1
E2 = kA2 2
2
1
= kA12
2
= E1

Since, linear momentum is conserved in both the cases,


Hence, in case 1,
( M + m ) v2 =
Mv1
 M 
v2 =   v1
M +m

Since,
M +m> M
M
<1
M +m

So,
v2 < v1

Advanced Paper-2
Hence, the speed is decreased when the masses are combined.
In case 2,
( M + m ) v2 =
Mv1
 M 
v2 =   v1
M +m

Since,
M +m> M
M
<1
M +m

So,
v2 < v1

Hence, the speed is decreased when the masses are combined.

13. Sol:
The S1S2 line is perpendicular to the screen which makes the
shape of the pattern concentric semicircle. To decide whether
the region is in dark zone or the bright zone use the following
formula at O,
2π 2π
= (φ ) ( S1O − S2O )
λ λ

−9 (
= 0.6003 ×10−3 )
600 ×10
= 2001π

Therefore, the region very close to the point O will be dark.

Advanced Paper-2
14. Sol:
(i) When the wire loop is entering in the field,

The formula to calculate force is,


F = BiL
dv B 2 L2 v
−mv =
dx R
2 2
v B L x
∫v0 dv =
mR ∫0
dx

 B 2 L2 
v= v0 −  x
 mR 

For right edge of the loop from x=0 to x=L

Current,
vBL
i=
R
  B 2 L2  
v −
 0   x  BL
  mR  
=
R
v BL B 3 L3
= 0 − x
R mR

And force,

Advanced Paper-2
 v0 BL B 3 L3 
 − x  BL
 R mR 
F=
R
2 2
v B L B 4 L4
= 0 2 − x
R mR 2

(ii)
When the loop is completely inside there is no current flow due
to uniform magnetic field.
So, i = 0 , v = and F =0

(iii)
When the loop is leaving,
For left edge of the loop from x = 3L to x = 4L

Current,
vBL
I= −
R
  B 2 L2  
− v
 0  +  x  BL
 mR  
=
R
v BL B 3 L3
=− 0 + x
R mR

And force,
 v0 BL B 3 L3 
− + x  BL
 R mR 
F=
R
2 2
v B L B 4 L4
= − 0 2 + x
R mR 2

15. Sol:
Advanced Paper-2
Acceleration is expressed as,
a = ω 2r
dv
v = ω 2r
dr
vdv = ω 2 rdr

Integrate,
v r
∫0
vdv = ∫ ω 2 rdr
R 2
r
v2  r2 
= ω2  
2  2 R 2

v2  r 2 − ( R2 4) 
=ω 
2

2  2 

=v ω r 2 − ( R2 4)

Velocity is defined by the expression,


dr
v=
dt
dr = vdt
=dr (ω )
r 2 − ( R 2 4 ) dt

Integrate,
r dr t
∫ = ∫ (ω ) dt
r 2 − ( R2 4)
R 2 0

R ωt
=r
4
( e + e −ωt )

16. Sol:
The expression for position is,
R ωt −ωt
=r
4
(e − e )
Advanced Paper-2
The velocity is,
dr
vrotational =
dt
ω R ωt −ωt
=
4
(e − e )

The rotational force is,


      
Frotational = Fin + 2m ( vrotational × ω ) + m (ω × r ) × ω
= ( )
−mω 2 riˆ + 2mvrotationalω − ˆj + mω 2 riˆ
= −2mω rrotational ˆj
ω 2 R ωt −ωt ˆ
=

2
(e − e ) j
The net force is,
  
Fnet =
− Frotational + Fg
ω2R
=
2
(eωt
− e −ωt ) ˆj + mgkˆ

17. Sol:
The ball is acted upon by electrostatic force in one direction,
thus option (A) cannot be correct.
After hitting the top plate, the balls will get negatively charged
and will now get attracted to the positively charged bottom
plate. Hence, option (B) is incorrect while option (C) is correct.
The motion of the balls is will be periodic because of
continuous change in nature of the charges. But it won’t be

Advanced Paper-2
simple harmonic motion because the motion described here
does not obey the principle of simple harmonic motion,
d2x
∝ −x
dt 2

Hence option (D) is incorrect.

18. Sol:
For the given system of electrostatic charges,
q ∝ V0
q = CV0

The formula for the distance travelled by the ball is,


1 2
h= at
2
1  qE  2
=  t
2 m 

Since,
dV
E= −
dy

= −
( −V0 ) − (V0 )
h
2V0
=
h

So,
  2V  
 CV0  0  
1  h  t2
h= 
2 m 
 
 
2
CV0 2
h= t
mh

Advanced Paper-2
Thus,
1
t2 ∝
V02
1
t∝
V0

Average current,
q
I avg =
t
I avg ∝ V02

Advanced Paper-2

You might also like